Derivative and Jacobian of a transformation

Click For Summary
The discussion revolves around calculating the derivative and Jacobian for the transformation defined by x = rcos(θ) and y = rsin(θ). Participants express confusion about which derivative to compute since both x and y depend on r and θ, emphasizing the need for partial derivatives. The Jacobian matrix is identified as the matrix of these partial derivatives, and its determinant represents the Jacobian. A reference to the Jacobian Wikipedia page is mentioned for further clarification. The conversation concludes with an acknowledgment of the relationship between small displacements and the Jacobian, akin to the chain rule.
ElijahRockers
Gold Member
Messages
260
Reaction score
10

Homework Statement



For the transformation, draw the lattice lines, calculate the derivative, and calculate the Jacobian.

x=rcos\theta
y=rsin\theta

The Attempt at a Solution



I drew the lattice lines correctly. What I am confused about is the derivative. Since x and y are both functions of r and theta, what derivative are they talking about? Wouldn't I have to take the partial with respect to R or theta? This section is supposed to be on calculating determinants.

I understand that the Jacobian is the determinant of a particular matrix, but where does this matrix come from?
 
Physics news on Phys.org
the Jacobian matrix is the matrix of partial derivatives
<br /> \begin{pmatrix}<br /> \frac{\partial x}{\partial r} &amp; \frac{\partial x}{\partial \theta} \\<br /> \frac{\partial y}{\partial r} &amp; \frac{\partial y}{\partial \theta} \\<br /> \end{pmatrix}<br />

the Jacobian determinant is the determinant of that matrix and is probably what you're referring to with the shorthand "Jacobian"
 
Alright thanks. Yea I looked at the Jacobian wikipedia earlier, before I posted this, but it all just seemed greek to me. I am about to try to finish the assignment.

Thanks again, I'll let you know how it goes.
 
Last edited:
note if you had a small displacement (dr,dtheta)^T, multiplying this by the jacobian would give you the corresponding (dx,dy), similar to the chain rule
 
Question: A clock's minute hand has length 4 and its hour hand has length 3. What is the distance between the tips at the moment when it is increasing most rapidly?(Putnam Exam Question) Answer: Making assumption that both the hands moves at constant angular velocities, the answer is ## \sqrt{7} .## But don't you think this assumption is somewhat doubtful and wrong?

Similar threads

  • · Replies 2 ·
Replies
2
Views
1K
  • · Replies 20 ·
Replies
20
Views
2K
  • · Replies 7 ·
Replies
7
Views
5K
  • · Replies 3 ·
Replies
3
Views
3K
  • · Replies 2 ·
Replies
2
Views
2K
  • · Replies 3 ·
Replies
3
Views
2K
  • · Replies 1 ·
Replies
1
Views
1K
Replies
3
Views
2K
  • · Replies 14 ·
Replies
14
Views
2K
  • · Replies 6 ·
Replies
6
Views
2K